Sie sind auf Seite 1von 2

DEPARTMENT OF MATHEMATICS, IIT GUWAHATI

MA101: Mathematics I Quiz I (Maximum Marks: 10)


Date: August 26, 2011 Time: 8 am - 8:50 am
Model Solutions
1. Let A be an n n matrix such that the system of equations Ax = 0 has a non-trivial solution. Is it possible that
the system of equations A
t
x = b has a unique solution for some b R
n
? Justify. 2
Solution: Since Ax = 0 has a non-trivial solution, we have rank(A) < n. We know that rank(A
t
) = rank(A),
and so rank(A
t
) < n. If rank[A
t
| b] = rank(A
t
), then the system A
t
x = b has no solution. If rank[A
t
| b] =
rank(A
t
) < n, then the system A
t
x = b will have innitely many solutions. In either case, A
t
x = b cannot have
a unique solution.
Aliter
Since the system of equations Ax = 0 has a non-trivial solution, we nd that the matrix A is not invertible. We
also know that A is invertible i A
t
is invertible. Thus the matrix A
t
is not invertible, and hence the system of
equations A
t
x = b cannot have a unique solution for any b R
n
.
2. Examine whether the following sets are subspaces of R
2
: 1+1
(a) {[x, y]
t
: x R, y Z} (b) {[x, y]
t
R
2
: 2x +y = 0 or x + 2y = 0}.
Solution: (a) Let S = {[x, y]
t
: x R, y Z}. We see that [0, 1]
t
S and
1
2
R, but
1
2
[0, 1]
t
=

0,
1
2

t
/ S.
Hence S is not a subspace of R
2
.
(b) Let T = {[x, y]
t
R
2
: 2x + y = 0 or x + 2y = 0}. We see that [1, 2]
t
T and [2, 1]
t
T, but
[1, 2]
t
+ [2, 1]
t
= [1, 1]
t
/ T. Hence T is not a subspace of R
2
.
3. Prove or disprove: If S = {v
1
, v
2
, v
3
, v
4
, v
5
, v
6
} is a linearly dependent set of vectors in R
5
then each vector of S
must be a linear combination of the remaining vectors of S. 2
Solution: Consider the set S = {v
1
, v
2
, v
3
, v
4
, v
5
, v
6
}, where v
1
= [1, 0, 0, 0, 0]
t
, v
2
= [0, 1, 0, 0, 0]
t
, v
3
=
[0, 0, 1, 0, 0]
t
, v
4
= [0, 0, 0, 1, 0]
t
, v
5
= [0, 0, 0, 0, 1]
t
and v
6
= 0 = [0, 0, 0, 0, 0]
t
. Clearly S is a linearly dependent
set, since 0 S. However, the vectors v
1
, v
2
, v
3
, v
4
and v
5
are linearly independent and v
6
= 0. Hence v
1
is not
a linear combination of the vectors v
2
, v
3
, v
4
, v
5
and v
6
.
Thus, the given statement is disproved by the above counterexample.
4. Let A and B be two matrices of sizes mn and n m respectively and let m > n. Prove that the matrix AB is
not invertible. 2
Solution: We have rank(AB) rank(B) n < m. Since AB is an m m matrix and rank(AB) < m, we nd
that the matrix AB is not invertible.
Aliter
Since rank(B) n < m, we have that there is non-zero solution of the system Bx = 0. Let u R
m
be such that
u = 0 and Bu = 0. Then we have (AB)u = A(Bu) = A0 = 0. Thus u is also a non-zero solution of the system
(AB)x = 0. Hence the matrix AB is not invertible.
5. Find a basis for the subspace spanned by the vectors [2, 1, 3, 1]
t
, [1, 2, 0, 1]
t
, [0, 2, 2, 1]
t
and [1, 1, 1, 1]
t
. 2
Solution: We have

1 2 0 1
2 1 3 1
0 2 2 1
1 1 1 1

R
2
R
2
2R
1

1 2 0 1
0 3 3 1
0 2 2 1
1 1 1 1

R
4
R
4
R
1

1 2 0 1
0 3 3 1
0 2 2 1
0 1 1 0

R
2

1
3
R
2

1 2 0 1
0 1 1
1
3
0 2 2 1
0 1 1 0

R
3
R
3
+ 2R
2

1 2 0 1
0 1 1
1
3
0 0 0
1
3
0 1 1 0

R
4
R
4
+R
2

1 2 0 1
0 1 1
1
3
0 0 0
1
3
0 0 0
1
3

R
4
R
4
+R
3

1 2 0 1
0 1 1
1
3
0 0 0
1
3
0 0 0 0

The last matrix is in row-echelon form. Hence a basis for the subspace spanned by the given vectors is

[1, 2, 0, 1]
t
, [0, 1, 1,
1
3
]
t
, [0, 0, 0,
1
3
]
t

.
End

Das könnte Ihnen auch gefallen